What's wrong with my proof of quotient rings?












1












$begingroup$


I seem to have arrived at a contradiction by applying what I know about quotient rings. I can't figure out where the mistake is.



Let $f(x)in mathbb{Z}[x]$, $deg f(x)ge 1$, and $p$ a prime number. Then



begin{align*}
mathbb{Z}[x]/f(x)mathbb{Z}[x]&cong (mathbb{Z}[x]/pmathbb{Z}[x])/((f(x)mathbb{Z}[x])/pmathbb{Z}[x])tag{Third Iso. Thm}\
&cong(mathbb{Z}[x]/pmathbb{Z}[x])/(hat{f}(x)(mathbb{Z}[x]/pmathbb{Z}[x]))tag{$*$}\
&cong mathbb{Z}_p[x]/hat{f}(x)mathbb{Z}_p[x].
end{align*}



where $(*)$ comes from:



begin{align*}
(f(x)mathbb{Z}[x])/pmathbb{Z}[x]&={g(x)+pmathbb{Z}[x]:g(x)in f(x)mathbb{Z}[x]}\
&={f(x)g(x)+pmathbb{Z}[x]:g(x)inmathbb{Z}[x]}\
&={(f(x)+pmathbb{Z}[x])(g(x)+pmathbb{Z}[x]):g(x)inmathbb{Z}[x]}\
&={hat{f}(x)(g(x)+pmathbb{Z}[x]):g(x)inmathbb{Z}[x]}\
&=hat{f}(x)(mathbb{Z}[x]/pmathbb{Z}[x])
end{align*}



For example, this would seem to show that $$mathbb{Z}[i]cong mathbb{Z}[x]/(x^2+1)mathbb{Z}[x]cong mathbb{Z}_2[x]/(x^2+1)mathbb{Z}_2[x]$$ The first ring is infinite and the last ring is finite. What's going on here?










share|cite|improve this question









$endgroup$












  • $begingroup$
    The problem is that $(Bbb Z[x])/(pBbb Z[x])$ is not equal to $(Bbb Z/p Bbb Z)[x]$
    $endgroup$
    – Omnomnomnom
    Jan 19 at 0:24










  • $begingroup$
    @Omnomnomnom Are you sure about that? Could you prove it in an answer form?
    $endgroup$
    – Riley
    Jan 19 at 0:32










  • $begingroup$
    @Omnomnomnom If you define $f:mathbb{Z}[x]to mathbb{Z}_p[x]$ by modding the coefficients by $p$, then isn't $text{Ker}(f)$ just $pmathbb{Z}[x]$, proving the desired isomorphism?
    $endgroup$
    – Riley
    Jan 19 at 0:42












  • $begingroup$
    @Riley An element in the ring $;left(Bbb Z/pBbb Zright)[x];$ is a polynomial, whereas an element in the ring $;Bbb Z[x]/pBbb Z[x];$ is not a polynomial but in fact an equivalence class...
    $endgroup$
    – DonAntonio
    Jan 19 at 0:43








  • 2




    $begingroup$
    Whatever you meant, the very first equality (isomorphism) is wrong unless $;pBbb Z[x]subset f(x)Bbb Z[x];$ , as required by the third isomorphism theorem.
    $endgroup$
    – DonAntonio
    Jan 19 at 0:49
















1












$begingroup$


I seem to have arrived at a contradiction by applying what I know about quotient rings. I can't figure out where the mistake is.



Let $f(x)in mathbb{Z}[x]$, $deg f(x)ge 1$, and $p$ a prime number. Then



begin{align*}
mathbb{Z}[x]/f(x)mathbb{Z}[x]&cong (mathbb{Z}[x]/pmathbb{Z}[x])/((f(x)mathbb{Z}[x])/pmathbb{Z}[x])tag{Third Iso. Thm}\
&cong(mathbb{Z}[x]/pmathbb{Z}[x])/(hat{f}(x)(mathbb{Z}[x]/pmathbb{Z}[x]))tag{$*$}\
&cong mathbb{Z}_p[x]/hat{f}(x)mathbb{Z}_p[x].
end{align*}



where $(*)$ comes from:



begin{align*}
(f(x)mathbb{Z}[x])/pmathbb{Z}[x]&={g(x)+pmathbb{Z}[x]:g(x)in f(x)mathbb{Z}[x]}\
&={f(x)g(x)+pmathbb{Z}[x]:g(x)inmathbb{Z}[x]}\
&={(f(x)+pmathbb{Z}[x])(g(x)+pmathbb{Z}[x]):g(x)inmathbb{Z}[x]}\
&={hat{f}(x)(g(x)+pmathbb{Z}[x]):g(x)inmathbb{Z}[x]}\
&=hat{f}(x)(mathbb{Z}[x]/pmathbb{Z}[x])
end{align*}



For example, this would seem to show that $$mathbb{Z}[i]cong mathbb{Z}[x]/(x^2+1)mathbb{Z}[x]cong mathbb{Z}_2[x]/(x^2+1)mathbb{Z}_2[x]$$ The first ring is infinite and the last ring is finite. What's going on here?










share|cite|improve this question









$endgroup$












  • $begingroup$
    The problem is that $(Bbb Z[x])/(pBbb Z[x])$ is not equal to $(Bbb Z/p Bbb Z)[x]$
    $endgroup$
    – Omnomnomnom
    Jan 19 at 0:24










  • $begingroup$
    @Omnomnomnom Are you sure about that? Could you prove it in an answer form?
    $endgroup$
    – Riley
    Jan 19 at 0:32










  • $begingroup$
    @Omnomnomnom If you define $f:mathbb{Z}[x]to mathbb{Z}_p[x]$ by modding the coefficients by $p$, then isn't $text{Ker}(f)$ just $pmathbb{Z}[x]$, proving the desired isomorphism?
    $endgroup$
    – Riley
    Jan 19 at 0:42












  • $begingroup$
    @Riley An element in the ring $;left(Bbb Z/pBbb Zright)[x];$ is a polynomial, whereas an element in the ring $;Bbb Z[x]/pBbb Z[x];$ is not a polynomial but in fact an equivalence class...
    $endgroup$
    – DonAntonio
    Jan 19 at 0:43








  • 2




    $begingroup$
    Whatever you meant, the very first equality (isomorphism) is wrong unless $;pBbb Z[x]subset f(x)Bbb Z[x];$ , as required by the third isomorphism theorem.
    $endgroup$
    – DonAntonio
    Jan 19 at 0:49














1












1








1





$begingroup$


I seem to have arrived at a contradiction by applying what I know about quotient rings. I can't figure out where the mistake is.



Let $f(x)in mathbb{Z}[x]$, $deg f(x)ge 1$, and $p$ a prime number. Then



begin{align*}
mathbb{Z}[x]/f(x)mathbb{Z}[x]&cong (mathbb{Z}[x]/pmathbb{Z}[x])/((f(x)mathbb{Z}[x])/pmathbb{Z}[x])tag{Third Iso. Thm}\
&cong(mathbb{Z}[x]/pmathbb{Z}[x])/(hat{f}(x)(mathbb{Z}[x]/pmathbb{Z}[x]))tag{$*$}\
&cong mathbb{Z}_p[x]/hat{f}(x)mathbb{Z}_p[x].
end{align*}



where $(*)$ comes from:



begin{align*}
(f(x)mathbb{Z}[x])/pmathbb{Z}[x]&={g(x)+pmathbb{Z}[x]:g(x)in f(x)mathbb{Z}[x]}\
&={f(x)g(x)+pmathbb{Z}[x]:g(x)inmathbb{Z}[x]}\
&={(f(x)+pmathbb{Z}[x])(g(x)+pmathbb{Z}[x]):g(x)inmathbb{Z}[x]}\
&={hat{f}(x)(g(x)+pmathbb{Z}[x]):g(x)inmathbb{Z}[x]}\
&=hat{f}(x)(mathbb{Z}[x]/pmathbb{Z}[x])
end{align*}



For example, this would seem to show that $$mathbb{Z}[i]cong mathbb{Z}[x]/(x^2+1)mathbb{Z}[x]cong mathbb{Z}_2[x]/(x^2+1)mathbb{Z}_2[x]$$ The first ring is infinite and the last ring is finite. What's going on here?










share|cite|improve this question









$endgroup$




I seem to have arrived at a contradiction by applying what I know about quotient rings. I can't figure out where the mistake is.



Let $f(x)in mathbb{Z}[x]$, $deg f(x)ge 1$, and $p$ a prime number. Then



begin{align*}
mathbb{Z}[x]/f(x)mathbb{Z}[x]&cong (mathbb{Z}[x]/pmathbb{Z}[x])/((f(x)mathbb{Z}[x])/pmathbb{Z}[x])tag{Third Iso. Thm}\
&cong(mathbb{Z}[x]/pmathbb{Z}[x])/(hat{f}(x)(mathbb{Z}[x]/pmathbb{Z}[x]))tag{$*$}\
&cong mathbb{Z}_p[x]/hat{f}(x)mathbb{Z}_p[x].
end{align*}



where $(*)$ comes from:



begin{align*}
(f(x)mathbb{Z}[x])/pmathbb{Z}[x]&={g(x)+pmathbb{Z}[x]:g(x)in f(x)mathbb{Z}[x]}\
&={f(x)g(x)+pmathbb{Z}[x]:g(x)inmathbb{Z}[x]}\
&={(f(x)+pmathbb{Z}[x])(g(x)+pmathbb{Z}[x]):g(x)inmathbb{Z}[x]}\
&={hat{f}(x)(g(x)+pmathbb{Z}[x]):g(x)inmathbb{Z}[x]}\
&=hat{f}(x)(mathbb{Z}[x]/pmathbb{Z}[x])
end{align*}



For example, this would seem to show that $$mathbb{Z}[i]cong mathbb{Z}[x]/(x^2+1)mathbb{Z}[x]cong mathbb{Z}_2[x]/(x^2+1)mathbb{Z}_2[x]$$ The first ring is infinite and the last ring is finite. What's going on here?







abstract-algebra ring-theory fake-proofs






share|cite|improve this question













share|cite|improve this question











share|cite|improve this question




share|cite|improve this question










asked Jan 19 at 0:18









RileyRiley

2,1551420




2,1551420












  • $begingroup$
    The problem is that $(Bbb Z[x])/(pBbb Z[x])$ is not equal to $(Bbb Z/p Bbb Z)[x]$
    $endgroup$
    – Omnomnomnom
    Jan 19 at 0:24










  • $begingroup$
    @Omnomnomnom Are you sure about that? Could you prove it in an answer form?
    $endgroup$
    – Riley
    Jan 19 at 0:32










  • $begingroup$
    @Omnomnomnom If you define $f:mathbb{Z}[x]to mathbb{Z}_p[x]$ by modding the coefficients by $p$, then isn't $text{Ker}(f)$ just $pmathbb{Z}[x]$, proving the desired isomorphism?
    $endgroup$
    – Riley
    Jan 19 at 0:42












  • $begingroup$
    @Riley An element in the ring $;left(Bbb Z/pBbb Zright)[x];$ is a polynomial, whereas an element in the ring $;Bbb Z[x]/pBbb Z[x];$ is not a polynomial but in fact an equivalence class...
    $endgroup$
    – DonAntonio
    Jan 19 at 0:43








  • 2




    $begingroup$
    Whatever you meant, the very first equality (isomorphism) is wrong unless $;pBbb Z[x]subset f(x)Bbb Z[x];$ , as required by the third isomorphism theorem.
    $endgroup$
    – DonAntonio
    Jan 19 at 0:49


















  • $begingroup$
    The problem is that $(Bbb Z[x])/(pBbb Z[x])$ is not equal to $(Bbb Z/p Bbb Z)[x]$
    $endgroup$
    – Omnomnomnom
    Jan 19 at 0:24










  • $begingroup$
    @Omnomnomnom Are you sure about that? Could you prove it in an answer form?
    $endgroup$
    – Riley
    Jan 19 at 0:32










  • $begingroup$
    @Omnomnomnom If you define $f:mathbb{Z}[x]to mathbb{Z}_p[x]$ by modding the coefficients by $p$, then isn't $text{Ker}(f)$ just $pmathbb{Z}[x]$, proving the desired isomorphism?
    $endgroup$
    – Riley
    Jan 19 at 0:42












  • $begingroup$
    @Riley An element in the ring $;left(Bbb Z/pBbb Zright)[x];$ is a polynomial, whereas an element in the ring $;Bbb Z[x]/pBbb Z[x];$ is not a polynomial but in fact an equivalence class...
    $endgroup$
    – DonAntonio
    Jan 19 at 0:43








  • 2




    $begingroup$
    Whatever you meant, the very first equality (isomorphism) is wrong unless $;pBbb Z[x]subset f(x)Bbb Z[x];$ , as required by the third isomorphism theorem.
    $endgroup$
    – DonAntonio
    Jan 19 at 0:49
















$begingroup$
The problem is that $(Bbb Z[x])/(pBbb Z[x])$ is not equal to $(Bbb Z/p Bbb Z)[x]$
$endgroup$
– Omnomnomnom
Jan 19 at 0:24




$begingroup$
The problem is that $(Bbb Z[x])/(pBbb Z[x])$ is not equal to $(Bbb Z/p Bbb Z)[x]$
$endgroup$
– Omnomnomnom
Jan 19 at 0:24












$begingroup$
@Omnomnomnom Are you sure about that? Could you prove it in an answer form?
$endgroup$
– Riley
Jan 19 at 0:32




$begingroup$
@Omnomnomnom Are you sure about that? Could you prove it in an answer form?
$endgroup$
– Riley
Jan 19 at 0:32












$begingroup$
@Omnomnomnom If you define $f:mathbb{Z}[x]to mathbb{Z}_p[x]$ by modding the coefficients by $p$, then isn't $text{Ker}(f)$ just $pmathbb{Z}[x]$, proving the desired isomorphism?
$endgroup$
– Riley
Jan 19 at 0:42






$begingroup$
@Omnomnomnom If you define $f:mathbb{Z}[x]to mathbb{Z}_p[x]$ by modding the coefficients by $p$, then isn't $text{Ker}(f)$ just $pmathbb{Z}[x]$, proving the desired isomorphism?
$endgroup$
– Riley
Jan 19 at 0:42














$begingroup$
@Riley An element in the ring $;left(Bbb Z/pBbb Zright)[x];$ is a polynomial, whereas an element in the ring $;Bbb Z[x]/pBbb Z[x];$ is not a polynomial but in fact an equivalence class...
$endgroup$
– DonAntonio
Jan 19 at 0:43






$begingroup$
@Riley An element in the ring $;left(Bbb Z/pBbb Zright)[x];$ is a polynomial, whereas an element in the ring $;Bbb Z[x]/pBbb Z[x];$ is not a polynomial but in fact an equivalence class...
$endgroup$
– DonAntonio
Jan 19 at 0:43






2




2




$begingroup$
Whatever you meant, the very first equality (isomorphism) is wrong unless $;pBbb Z[x]subset f(x)Bbb Z[x];$ , as required by the third isomorphism theorem.
$endgroup$
– DonAntonio
Jan 19 at 0:49




$begingroup$
Whatever you meant, the very first equality (isomorphism) is wrong unless $;pBbb Z[x]subset f(x)Bbb Z[x];$ , as required by the third isomorphism theorem.
$endgroup$
– DonAntonio
Jan 19 at 0:49










1 Answer
1






active

oldest

votes


















1












$begingroup$

Whatever you meant, the very first equality (isomorphism) is wrong unless $;pBbb Z[x]⊂f(x)Bbb Z[x];$ , as required by the third isomorphism theorem






share|cite|improve this answer









$endgroup$













    Your Answer





    StackExchange.ifUsing("editor", function () {
    return StackExchange.using("mathjaxEditing", function () {
    StackExchange.MarkdownEditor.creationCallbacks.add(function (editor, postfix) {
    StackExchange.mathjaxEditing.prepareWmdForMathJax(editor, postfix, [["$", "$"], ["\\(","\\)"]]);
    });
    });
    }, "mathjax-editing");

    StackExchange.ready(function() {
    var channelOptions = {
    tags: "".split(" "),
    id: "69"
    };
    initTagRenderer("".split(" "), "".split(" "), channelOptions);

    StackExchange.using("externalEditor", function() {
    // Have to fire editor after snippets, if snippets enabled
    if (StackExchange.settings.snippets.snippetsEnabled) {
    StackExchange.using("snippets", function() {
    createEditor();
    });
    }
    else {
    createEditor();
    }
    });

    function createEditor() {
    StackExchange.prepareEditor({
    heartbeatType: 'answer',
    autoActivateHeartbeat: false,
    convertImagesToLinks: true,
    noModals: true,
    showLowRepImageUploadWarning: true,
    reputationToPostImages: 10,
    bindNavPrevention: true,
    postfix: "",
    imageUploader: {
    brandingHtml: "Powered by u003ca class="icon-imgur-white" href="https://imgur.com/"u003eu003c/au003e",
    contentPolicyHtml: "User contributions licensed under u003ca href="https://creativecommons.org/licenses/by-sa/3.0/"u003ecc by-sa 3.0 with attribution requiredu003c/au003e u003ca href="https://stackoverflow.com/legal/content-policy"u003e(content policy)u003c/au003e",
    allowUrls: true
    },
    noCode: true, onDemand: true,
    discardSelector: ".discard-answer"
    ,immediatelyShowMarkdownHelp:true
    });


    }
    });














    draft saved

    draft discarded


















    StackExchange.ready(
    function () {
    StackExchange.openid.initPostLogin('.new-post-login', 'https%3a%2f%2fmath.stackexchange.com%2fquestions%2f3078901%2fwhats-wrong-with-my-proof-of-quotient-rings%23new-answer', 'question_page');
    }
    );

    Post as a guest















    Required, but never shown

























    1 Answer
    1






    active

    oldest

    votes








    1 Answer
    1






    active

    oldest

    votes









    active

    oldest

    votes






    active

    oldest

    votes









    1












    $begingroup$

    Whatever you meant, the very first equality (isomorphism) is wrong unless $;pBbb Z[x]⊂f(x)Bbb Z[x];$ , as required by the third isomorphism theorem






    share|cite|improve this answer









    $endgroup$


















      1












      $begingroup$

      Whatever you meant, the very first equality (isomorphism) is wrong unless $;pBbb Z[x]⊂f(x)Bbb Z[x];$ , as required by the third isomorphism theorem






      share|cite|improve this answer









      $endgroup$
















        1












        1








        1





        $begingroup$

        Whatever you meant, the very first equality (isomorphism) is wrong unless $;pBbb Z[x]⊂f(x)Bbb Z[x];$ , as required by the third isomorphism theorem






        share|cite|improve this answer









        $endgroup$



        Whatever you meant, the very first equality (isomorphism) is wrong unless $;pBbb Z[x]⊂f(x)Bbb Z[x];$ , as required by the third isomorphism theorem







        share|cite|improve this answer












        share|cite|improve this answer



        share|cite|improve this answer










        answered Jan 19 at 1:17









        DonAntonioDonAntonio

        179k1494230




        179k1494230






























            draft saved

            draft discarded




















































            Thanks for contributing an answer to Mathematics Stack Exchange!


            • Please be sure to answer the question. Provide details and share your research!

            But avoid



            • Asking for help, clarification, or responding to other answers.

            • Making statements based on opinion; back them up with references or personal experience.


            Use MathJax to format equations. MathJax reference.


            To learn more, see our tips on writing great answers.




            draft saved


            draft discarded














            StackExchange.ready(
            function () {
            StackExchange.openid.initPostLogin('.new-post-login', 'https%3a%2f%2fmath.stackexchange.com%2fquestions%2f3078901%2fwhats-wrong-with-my-proof-of-quotient-rings%23new-answer', 'question_page');
            }
            );

            Post as a guest















            Required, but never shown





















































            Required, but never shown














            Required, but never shown












            Required, but never shown







            Required, but never shown

































            Required, but never shown














            Required, but never shown












            Required, but never shown







            Required, but never shown







            Popular posts from this blog

            Can a sorcerer learn a 5th-level spell early by creating spell slots using the Font of Magic feature?

            ts Property 'filter' does not exist on type '{}'

            mat-slide-toggle shouldn't change it's state when I click cancel in confirmation window